Jump to content

Zet

Senior Members
  • Posts

    57
  • Joined

  • Last visited

Profile Information

  • Favorite Area of Science
    physics

Zet's Achievements

Meson

Meson (3/13)

-2

Reputation

  1. I get it. I was wrong. (Well, I don't totally get it. I need to spend some time studying this area of physics. But I get, I think, the basic overall logic of it.) So, when we have two electromagnets attracted to one another (and they have been for some time) the "center of energy density" is halfway between them (assuming the two electromagnets are the same). They are accelerating towards one another. One is then turned off. It stops accelerating. But the other one (the still turned on one) continues to accelerate for a while. This increased amount of momentum of the one electromagnet in one direction is offset by the "center of energy density" moving in the opposite direction towards this still accelerating magnet (the still turned on magnet). And so increased motion in one direction is matched by increased motion in the other direction. The increased mass times velocity of the accelerating electromagnet is equally matched by the mass times velocity of the moving "center of energy density" (and the decreasing amount of energy density) in the opposite direction. If I've stated this right, then I get it. ("Get it" is the broadest of meanings.) Cool. Thank you. I realize my snootiness is not forgivable, but I do apologize. Take care!
  2. . Yes, the supplicant has the burden of proof and not the pedagogue, if he is trying to prove something. However, if the supplicant asks a question and the pedagogue does not directly answer the question but rather points to a field of study, then, presumably, the teacher is saying that the answer lays in that area of study, and simply leaves it to the student to put the pieces together. So, in the thought experiment above where there are two electromagnets and they are turned on and off in such a way that the one is set into greater motion towards the other, if the response to “how is momentum is conserved?” is “electromagnetic fields have momentum” then presumably the answer lays in this aspect of physics (and it is just up to the student to put the pieces together). So, we have the one mass (the one electromagnet) set into more motion in one direction, and if momentum is conserved because “electromagnetic fields have momentum” then this must mean that the electromagnetic field moving towards this now moving magnet speeds up to offset the magnet’s increased velocity. But this would mean that the electromagnetic field is now moving faster than the speed of light. And it cannot. So, yes, this tangent was ignored in the statement of the thought experiment. But noting this absence, and noting it in such a way as to suggest that in it lays the answer to how this law is not violated is misleading at best. It appears to answer, or it appears to point to an answer, to the question but it does not. And this should then be called out. Yes? No? When the one electromagnet is set into more motion than the equal mass of the other electromagnet, how is Newton’s Third Law not violated? ? .
  3. There is nothing in your last post that I disagree with. However, there is nothing in you last post that addresses the issue. No one is asking you to give us a MOOC on electromagnetic theory. The issue raised is whether or not the third law of motion is violated in this case. And no one is even asking you in particular to address this either. However, if you seemly purport to have addressed it and seemly purport to have put the matter to rest, but you actually don't, then you are going to get called out on it. You seem to conclude with the speed of changes in a magnetic field in a vacuum transmit at the speed of light ( c ). Okay? Again, I don't disagree with you, but what does this have to do with addressing the issue? As far as I can tell, this is not a conclusion as to why there is not a violation, but rather this is a restatement of an initial premise.
  4. "Newton's third law is only seemingly violated in such situations. You need to remember that the electromagnetic field itself carries momentum." - ajb Okay. So make your case. Assertions are like opinions ... everybody's got one.
  5. . hey moth ... I believe I understand your point. In a closed system, the amount of energy in that system, is frame dependent. If you move from one inertial frame of reference to another, say in a system such as the Universe, the total amount of energy within that system will change. And so if you look at things from the faster moving magnet at rest and from the slower moving magnet at rest, you will get two different amounts of total energies between the two systems. Is this what you’re getting at? If so, I tried to address this in endnote #9 (but, again, the endnotes are superfluous to the question). Let me know what you think. And thank you for already telling me some of what you think! - Zet (PS: I am mistrustful of large font arguments too. That’s why I consider post #24 to be a failed “reductio ad absurdum with font formatting” attempt. At no time have I enlarged my fonts; only diminished them and grayed them. I simply offered to enlarge my fonts, if that is what he was complaining about in post #24. I don’t know.) .
  6. . I don’t. My original post is an easy read but it is very long. And I totally understand the idea that no one wants to waste their time reading that much text “knowing” that there must be some simple flaw (or flaws) buried somewhere in all that text. I get it. That’s why I spent time working with the issue to present it in another way and in a much shorter form (post #21). I don’t see how anyone could read your post #24 as anything other than a complaint about my use of fonts and their formatting. If there is another hidden implication in your single sentence response, I missed it. If anyone (“people”) other than John Cuthber is reading this, do you also find the text of the question in post #21, as written, something that has to be “struggled” with in order to get through? I’d like to know. I tried to make the issue as simple as possible. And I had believed that I had succeeded in this in post #21. But, now, John Cuthber is making the claim that the language and/or concepts are not clear in post #21 and if any of you others in the forum have attempted to understand it it was a “struggle” for you. Others, please let me know if this “struggle” is, in fact, also true for you. Thank you. - Zet .
  7. . I guess I would characterize your response as “reductio ad absurdum with font formatting.” That’s it? Am I to understand that the problem with my question in post #21 is not logical or factual but rather “an improper use of fonts”? The endnotes in post #21 are superfluous to the question and so are accordingly diminished in size. They are merely there in anticipation of the kinds of tangential issues that may arise when addressing the question. The actual question itself should be easily readable. I realize the original post in this thread is very long and probably no one (or hardly anyone) read it. But the reformulated question in post #21 (sans endnotes) only takes a couple of minutes to read. Does anyone have anything to say about the substance of the issue raised in post #21? (BTW: I can repost the text of post #21 in a large plain format such as 18 point Courier New. Would you like me to do so?) .
  8. . Silence, generally, means consent. “Silence implies consent” http://c2.com/cgi/wiki?SilenceImpliesConsent “he who is silent is taken to agree” http://en.wikipedia.org/wiki/Silence_procedure “Silence means consent” https://www.englishclub.com/ref/esl/Sayings/S/Silence_means_consent_925.htm “Silence gives consent” http://idioms.thefreedictionary.com/Silence+gives+consent Can I take the collective silence of the members of Science Forums to mean that I found a violation of the Law of Conservation of Energy in post #21? (I realize my original post in this thread is very long, but I worked hard to present the issue in a much shorter form in post #21 (especially if you just look at the question and ignore the endnotes).) Or, is my question in post #21 so embarrassingly stupid that it’s not worth anyone’s time to comment? Or, something else? ? .
  9. . Hello. If anyone has the time, could you please show me why the question in post #21 does not lead to a violation of the Law of Conservation of Energy? Thank you. - Zet I’m always willing to learn. I worked hard on the question. .
  10. . (Please note: I posted this question as its own new separate thread ... I thought it was distinct enough to warrant a new conversation ... I was wrong ... I misjudged the parameters of the rules of this forum ... I apologize ... I hope that I am now able/allowed to post it here, in this thread, even though I've already posted it in its own separate (and locked) thread ... thank you ... and, again, I apologize for my misjudgment.) There is an even simpler form of this question; with demagnetized and moving magnets. There are two closed systems. In terms of energy each system is identical. In each system, there are two magnetically aligned ferromagnets. One is fixed in place, and the other one is in motion moving towards the fixed one due to their mutual attraction. In each system, there is a chemical heat pack. If the chemicals are exposed to one another thermal energy will be generated. The temperature in the vicinity of the heat pack after the chemical exposure will be greater than the Curie temperature of the ferromagnets. (And after that, the thermal energy will then dissipate throughout the rest of the system.) The chemical heat pack is placed at one location in the one system and in the other system the chemical heat pack is placed near the fixed magnet. In both systems, when the two magnets in the two sets are the same distance apart, and when the moving magnet is near the heat pack in the one system, the chemicals are exposed to one another. The temperature in the vicinity of heat pack increases to more than the Curie temperature of the ferromagnets. One magnet is each system becomes demagnetized. (The magnets are close enough to one another to be attracted to one another (even if only slightly) while far enough away from one another so that the magnetic field of the other non-demagnetized magnet is not strong enough to keep the heated magnet externally significantly magnetically aligned.) There is a of loss mutual attraction between the two magnets. The remaining amount of potential energy between them as they have gotten closer is now gone. The demagnetized magnet is immediately cut off from the mutual attraction. But the lack of mutual attraction takes some time to make its way across the distance to the still magnetized magnet. It remains attracted to the other magnet for a while longer. The means in the case where it is the moving magnet that is demagnetized the moving demagnetized magnet immediately ceases to continue to accelerate, while in the other case where it is the fixed magnet that is demagnetized the moving still magnetized magnet continues to accelerate for a little while longer. In the end, in both systems, the thermal energy will dissipate throughout the entire closed system and both magnets within it will be raised above their Curie temperatures. The two moving magnets, in the two different systems, will continue to move. In the end the two systems are identical in many respects. There is the same decrease in chemical potential energy. There is the same increase in energy in the form of the magnets’ demagnetizations. And there is an increase in thermal energy. In the one system, there is, however, more kinetic energy in the end than in the other. And since both systems started out identical energy-wise they must end up with identical total amounts of energy, according to the logic of the Law of Conservation of Energy. And so, in the end, in the case with more kinetic energy there must be less of another form of energy, and in the other case with less kinetic energy there must be more of this other form of energy. What is it? Or, is there a flaw in the logic of the “Laws of Physics” in this situation? ? --- Endnotes. 1: The demagnetized state is the higher energy state and the magnetized state is the lower energy state (but this is irrelevant to the issue presented here). 2: It is a loss in thermal energy that demagnetizes each magnet and thus raises each magnet’s energy state. 3: The only possibility I can see is, in the end, there must be less thermal energy in the system with more kinetic energy and more thermal energy in the system with less kinetic energy. This means it must take more energy to demagnetize the fixed magnet (a greater decrease in thermal energy) and where the moving magnet continues to accelerate, and less energy to demagnetize the moving magnet (a lesser decrease in thermal energy) and where the moving magnet immediately discontinues to accelerate. But if anything it seems as if the exact opposite should be true. As the moving magnet moves towards the fixed magnet the strength of the moving magnet’s field moves too. Whereas the moving magnet moves across positions where the strength of the fixed magnet is already there. And so, at the time of demagnetization of each different magnet in the two cases, when the distance between the two magnets in both sets is identical, the strength of the magnetic field from the fixed magnet on the moving magnet could be stronger than the strength of the magnetic field from the moving magnet on the fixed magnet. And it stands to reason that the stronger the external magnetic field is on the demagnetizing magnet the more energy it would take to demagnetize it. But this leads to the exact opposite conclusion needed. This leads to that it would then take more energy to demagnetize the moving magnet, which is also the case where there is less kinetic energy in the end, and so there would both be less kinetic energy and less thermal energy than in the other case. This only exacerbates the problem. 4: There is also an empirical answer, if one has the skills and resources to find it. However, regardless of what the empirical answer turns out to be, the logic must work first or no matter what the empirical answer turns up to be it will not be one where energy is conserved. 5: If the loss of mutual attraction does not take time to cross the distance between the two magnets and to the still magnetized magnet, but is rather instantaneous, then this would lead to all sorts of Special Theory of Relativity paradoxes. 6: The “demagnetized” magnet will never fully become magnetically disaligned. And so, the moving magnets will continue to accelerate after the “demagnetization” of itself or the other magnet. However the residual magnetic alignment and the remaining continued acceleration are so minuscule so as to be reasonably ignored. 7: In reality, no mutually attracted body is “fixed in place.” They both are moving towards one another. But if the “fixed in place” magnets in the two systems above are connected to a massive body such as planet Earth, then the acceleration of the “fixed in place” magnet and the associated massive body is so miniscule so as to be reasonably ignored. There is an equal increase in momentum in opposite directions between the “moving” magnet and the “fixed in place” magnet and associated massive body (p = mv), but the increase in kinetic energy of the “fixed in place” magnet and associated massive body is miniscule compared to the increase in kinetic energy of the “moving” magnet and so can be reasonably ignored (ke = ½mv2). (However, whether it is the “moving” magnetic that continues to accelerate for a time while the other demagnetized body and associated massive body does not or whether it is the “fixed in place” magnet” and associated massive body continues to accelerate for a while while the other demagnetized body does not, either way, there is also a possible violation of the Law of Conservation of Momentum.) 8: The fact that one magnet is in motion relative to the chemical heat pack and the other magnet is not is irrelevant unlike other aspects of magnetic movement, such as Lenz’ Law, where the nature of any relative motion is a factor. 9: When two bodies are in two different motions, such as the faster moving magnet in the one system versus the slower moving magnet in the other system, they are in two different frames. And the total amount of energy can vary between frames. However, the two frames in comparison here are the two “fixed in place” frames and not the frames from the perspective of the “moving” bodies at rest, and so the total amount of energies between the two systems considered are in the same frame and so must end up with the same total amounts of energy in each for the logic of the Law of Conservation of Energy to not be violated. .
  11. . A Question about the Logic of the Law of Conservation of Energy with Demagnetized and Moving Magnets. There are two closed systems. In terms of energy each system is identical. In each system, there are two magnetically aligned ferromagnets. One is fixed in place, and the other one is in motion moving towards the fixed one due to their mutual attraction. In each system, there is a chemical heat pack. If the chemicals are exposed to one another thermal energy will be generated. The temperature in the vicinity of the heat pack after the chemical exposure will be greater than the Curie temperature of the ferromagnets. (And after that, the thermal energy will then dissipate throughout the rest of the system.) The chemical heat pack is placed at one location in the one system and in the other system the chemical heat pack is placed near the fixed magnet. In both systems, when the two magnets in the two sets are the same distance apart, and when the moving magnet is near the heat pack in the one system, the chemicals are exposed to one another. The temperature in the vicinity of heat pack increases to more than the Curie temperature of the ferromagnets. One magnet is each system becomes demagnetized. (The magnets are close enough to one another to be attracted to one another (even if only slightly) while far enough away from one another so that the magnetic field of the other non-demagnetized magnet is not strong enough to keep the heated magnet externally significantly magnetically aligned.) There is a of loss mutual attraction between the two magnets. The remaining amount of potential energy between them as they have gotten closer is now gone. The demagnetized magnet is immediately cut off from the mutual attraction. But the lack of mutual attraction takes some time to make its way across the distance to the still magnetized magnet. It remains attracted to the other magnet for a while longer. The means in the case where it is the moving magnet that is demagnetized the moving demagnetized magnet immediately ceases to continue to accelerate, while in the other case where it is the fixed magnet that is demagnetized the moving still magnetized magnet continues to accelerate for a little while longer. In the end, in both systems, the thermal energy will dissipate throughout the entire closed system and both magnets within it will be raised above their Curie temperatures. The two moving magnets, in the two different systems, will continue to move. In the end the two systems are identical in many respects. There is the same decrease in chemical potential energy. There is the same increase in energy in the form of the magnets’ demagnetizations. And there is an increase in thermal energy. In the one system, there is, however, more kinetic energy in the end than in the other. And since both systems started out identical energy-wise they must end up with identical total amounts of energy, according to the logic of the Law of Conservation of Energy. And so, in the end, in the case with more kinetic energy there must be less of another form of energy, and in the other case with less kinetic energy there must be more of this other form of energy. What is it? Or, is there a flaw in the logic of the “Laws of Physics” in this situation? ? --- Endnotes. 1: The demagnetized state is the higher energy state and the magnetized state is the lower energy state (but this is irrelevant to the issue presented here). 2: It is a loss in thermal energy that demagnetizes each magnet and thus raises each magnet’s energy state. 3: The only possibility I can see is, in the end, there must be less thermal energy in the system with more kinetic energy and more thermal energy in the system with less kinetic energy. This means it must take more energy to demagnetize the fixed magnet (a greater decrease in thermal energy) and where the moving magnet continues to accelerate, and less energy to demagnetize the moving magnet (a lesser decrease in thermal energy) and where the moving magnet immediately discontinues to accelerate. But if anything it seems as if the exact opposite should be true. As the moving magnet moves towards the fixed magnet the strength of the moving magnet’s field moves too. Whereas the moving magnet moves across positions where the strength of the fixed magnet is already there. And so, at the time of demagnetization of each different magnet in the two cases, when the distance between the two magnets in both sets is identical, the strength of the magnetic field from the fixed magnet on the moving magnet could be stronger than the strength of the magnetic field from the moving magnet on the fixed magnet. And it stands to reason that the stronger the external magnetic field is on the demagnetizing magnet the more energy it would take to demagnetize it. But this leads to the exact opposite conclusion needed. This leads to that it would then take more energy to demagnetize the moving magnet, which is also the case where there is less kinetic energy in the end, and so there would both be less kinetic energy and less thermal energy than in the other case. This only exacerbates the problem. 4: There is also an empirical answer, if one has the skills and resources to find it. However, regardless of what the empirical answer turns out to be, the logic must work first or no matter what the empirical answer turns up to be it will not be one where energy is conserved. 5: If the loss of mutual attraction does not take time to cross the distance between the two magnets and to the still magnetized magnet, but is rather instantaneous, then this would lead to all sorts of Special Theory of Relativity paradoxes. 6: The “demagnetized” magnet will never fully become magnetically disaligned. And so, the moving magnets will continue to accelerate after the “demagnetization” of itself or the other magnet. However the residual magnetic alignment and the remaining continued acceleration are so minuscule so as to be reasonably ignored. 7: In reality, no mutually attracted body is “fixed in place.” They both are moving towards one another. But if the “fixed in place” magnets in the two systems above are connected to a massive body such as planet Earth, then the acceleration of the “fixed in place” magnet and the associated massive body is so miniscule so as to be reasonably ignored. There is an equal increase in momentum in opposite directions between the “moving” magnet and the “fixed in place” magnet and associated massive body (p = mv), but the increase in kinetic energy of the “fixed in place” magnet and associated massive body is miniscule compared to the increase in kinetic energy of the “moving” magnet and so can be reasonably ignored (ke = ½mv2). (However, whether it is the “moving” magnetic that continues to accelerate for a time while the other demagnetized body and associated massive body does not or whether it is the “fixed in place” magnet” and associated massive body continues to accelerate for a while while the other demagnetized body does not, either way, there is also a possible violation of the Law of Conservation of Momentum.) 8: The fact that one magnet is in motion relative to the chemical heat pack and the other magnet is not is irrelevant unlike other aspects of magnetic movement, such as Lenz’ Law, where the nature of any relative motion is a factor. 9: When two bodies are in two different motions, such as the faster moving magnet in the one system versus the slower moving magnet in the other system, they are in two different frames. And the total amount of energy can vary between frames. However, the two frames in comparison here are the two “fixed in place” frames and not the frames from the perspective of the “moving” bodies at rest, and so the total amount of energies between the two systems considered are in the same frame and so must end up with the same total amounts of energy in each for the logic of the Law of Conservation of Energy to not be violated. .
  12. . The demagnetized state is the higher energy state and magnetized state is the lower energy state. “... this torque tends to line up the magnetic moment with the magnetic field B, so this represents its lowest energy configuration” http://hyperphysics.phy-astr.gsu.edu/hbase/magnetic/magmom.html “... the energy is lowest when the magnetic moment is aligned with the magnetic field.” http://hyperphysics.phy-astr.gsu.edu/hbase/magnetic/magpot.html “This reduces the electrostatic energy of the electrons when their spins are parallel compared to their energy when the spins are anti-parallel, so the parallel-spin state is more stable.” http://en.wikipedia.org/wiki/Ferromagnetism “This "spin flip" places some of the spins in their higher energy state. If the radio frequency signal is then switched off, the relaxation of the spins back to the lower state ...” http://hyperphysics.phy-astr.gsu.edu/hbase/nuclear/nmr.html But this is irrelevant to the issue presented here. .
  13. Yeah. I tried to set up my thought experiment to avoid issues of “thrust” or “different amounts of initial kinetic energy” and just get to where I’m having a problem (different decreases in pressures on the top and bottom adding to the rise of the airfoils). I tried, ... but I don’t think I succeeded. Airfoils are complicated things, and I probably should stay away from them. I do thank you all for adding to my understanding of the issue. Whether you all believe me or not, I have learned from this discussion. Thank you for all of your time, effort, and ... patience!
  14. . So, when the fluid moves over the airfoil there is a certain amount of "dynamic" potential energy. If the airfoil is free to rise, it will. When it rises this potential energy becomes kinetic energy. Okay ... cool. What then happens to this "dynamic" potential energy when the airfoil does not rise ... and after the fluid passes and this potential energy is then gone? The Law of Conservation of Energy states that energy can change forms, but the total amount remains. Are you suggesting that there was an amount of energy in the form of "dynamic" potential energy in the non-rising case and then this amount of energy is then gone (without an offsetting change in another form of energy)? If so, you are suggesting a violation of the Law of Conservation of Energy. ?
×
×
  • Create New...

Important Information

We have placed cookies on your device to help make this website better. You can adjust your cookie settings, otherwise we'll assume you're okay to continue.